Series 7: Missed Questions

¡Supera tus tareas y exámenes ahora con Quizwiz!

Functional allocation

is a sharing arrangement in which the general partner pays for all tangible drilling costs (capitalized costs), and the limited partners pay for all intangible drilling costs (deductible costs).

Which of the following statements regarding Treasury bills are true? 1They are sold in minimum denominations of $10,000. 2They are offered with maturities ranging up to 52 weeks. 3Their interest is exempt from taxation at the state level. 4They are callable by the U.S. Treasury at any time before maturity.

2 & 3) Treasury bills are sold in minimum denominations of $100 and are not callable before maturity. T-bills are regularly offered with maturities from four weeks to as long as 52 weeks from issuance and are issued at a discount. Interest on Treasury bills is taxable at the federal level only.

Which of the following statements regarding Ginnie Maes are true? 1 They are quoted in 1/8ths. 2 They are quoted in 1/32nds. 3 They are traded with an accrued interest computed on an actual-day basis. 4 They are traded with an accrued interest computed on a 30/360 basis.

2 & 4

Which of the following order types are available to customers for use in NYSE equity markets? Fill or kill (FOK) Immediate or cancel (IOC) All or none (AON) Order cancels other (OCO) A) I and III B) II and IV C) II and III D) I and IV

2 and 4 B) IOC and OCO orders are available to customers for use in the NYSE equity markets. FOK and AON orders are no longer permitted in NYSE equity markets.

Which of the following statements are true? Build America Bonds (BABs) are tax exempt at all levels. Direct-payment BABs provide the municipal issuer with payments from the U.S. Treasury. BABs are issued by the U.S. Treasury. Tax credit or issuer BABs provide the municipal bondholder with a federal income tax credit. A) I and III B) I and IV C) II and IV D) II and III

2 and 4 Created under the Economic Recovery and Reinvestment Act of 2009 to assist in reducing costs to issuing municipalities and to stimulate the economy, BABs are taxable municipal securities. There are two types: direct payment BABs that provide the municipal issuer with payments from the U.S. Treasury and tax credit or issuer BABs that provide the bondholder with a federal income tax credit.

Which of the following would be considered in analyzing the credit worthiness of a revenue bond issuer? 1 Per capita debt 2 Debt service coverage 3Management 4 Debt to assessed valuation

2& 3 Revenue bonds are paid out of revenues from a particular project or facility, not from tax revenue. Therefore, debt service coverage and the personnel in charge of managing the facility are important. Overall debt of the issuer would be important in analyzing a general obligation bond backed by the issuer's full faith and credit.

Achieving a Better Life Experience (ABLE) accounts are tax-advantaged savings accounts for individuals with disabilities and their families. The ABLE Act limits eligibility to individuals with significant disabilities where the age of onset of the disability occurred before turning age A) 18. B) 30. C) 21. D) 26.

26One need not be under the age of 26 to be eligible to establish an ABLE account. One could be over the age of 26, but as long as the onset of the disability occurred before age 26, the person is eligible to establish an ABLE account.

The MSRB classifies municipal securities into two categories: notes and bonds. They define bonds as any municipal debt security with a maturity of A) 10 years or more. B) 5 years or more. C) 1 year or more. D) 2 years or more.

5 years Municipal notes have a maximum maturity of less than five years. Once the municipal security is issued with a maturity of 5 years or longer, it is considered a municipal bond.

Which of the following debt instruments would likely be suitable for sophisticated investors only? A) Equity-linked notes B) Jumbo CDs C) First mortgage bonds D) Debentures

A ) Despite the misleading name, ELNs are debt instruments. When traded on an exchange, they are exchange-traded notes (ETNs). In either case, these are considered alternative products with unique risks, and therefore, not suitable for most investors. Although debentures are corporate debt without any pledged collateral, some of the financially strongest companies in the country issue them and receive high ratings. Even though Jumbo CDs require a minimum of $100,000, it does not require any sophistication to understand the product.

A UK company exports sweaters to the U.S. and will be paid in U.S. dollars upon delivery. To hedge foreign-exchange risk using listed currency options, the UK company should A) buy British pound calls. B) buy British pound puts. C) sell British pound puts. D) sell British pound calls.

A ) Normally, exporters buy puts on foreign currency to hedge. There are no listed currency options available on the U.S. dollar, so the British company should buy calls on its own currency.

A new client, age 25, earning $41,000 annually has saved $20,000 to allocate to an investment portfolio for the first time. The client conveys that while he would like to see some growth, an investment with moderate risk and some downside protection are important objectives for his first time investing. Aligning with the client's investment experience and objectives, which of the following would be the most suitable? A) Municipal bond fund B) Balanced fund C) Equities index fund D) Money market fund

A balanced fund, which consists of both equities and debt instruments, not only aligns with the growth objective but also offers some downside protection against falls in market due to the debt portion of its portfolio. Equity index funds move with the markets and offer no downside protection. A money market fund would not align with growth, and a municipal bond fund would have no benefit for an investor in a lower income bracket.

Which of the following registers the securities and packages the program for a limited partnership? A) Property manager B) Limited partners C) Syndicator D) General partner

A syndicator handles the registration of the limited partnership units.

Which of the following statements regarding fixed municipal unit trusts are true? The trust is managed. The trust is not managed. The portfolio can be traded. The portfolio cannot be traded. A) II and IV B) I and IV C) II and III D) I and III

A) Fixed unit trusts are not managed; the portfolio of securities does not change. As bonds mature or are called, the proceeds are distributed pro rata to the unit holders. These units are redeemable by the issuer or its agent.

A customer is long 200 shares of MTN at 30 and 400 shares of DWQ at 20 in a margin account. If the debit balance in the account is $8,000, and the customer sells 200 DWQ shares for $4,000, the credit to special memorandum account (SMA) is A) $2,000. B) $4,000. C) $1,000. D) $0.

A) 2,000 Because this account is below 50% margin, the account is restricted ($6,000 equity divided by $14,000 market value equals 42.8% equity). When securities are sold in a restricted account, 50% of the proceeds are released to SMA. Because $4,000 worth of securities were sold, $2,000 (50%) is credited to SMA.

A customer buys a new issue municipal bond with a dated date of January 1 for settlement on January 31. If the first interest payment date is March 1, how many days of accrued interest will the customer pay to the syndicate? A) 30 B) 60 C) 0 D) 31

A) 30 In this new issue, interest begins to accrue as of the dated date, so the customer (buyer) must pay the syndicate interest from the dated date up to, but not including, the settlement date. The number of days from January 1 up to, but not including, January 31 is 30.

An instrument that illustrates the transfer of title to any dividend, interest, or right that pertains to securities that are contracted for is called A) a due bill. B) a right. C) a power of attorney. D) a warrant.

A) A due bill is an assignment of a forthcoming distribution from the seller to the new owner.

In the case of an unsolicited order, a prospectus must be delivered to the purchaser of a unit investment trust A) with the purchase confirmation. B) before the month's end. C) between 45 days and 18 months following the initial deposit. D) before the purchase.

A) A purchaser of newly issued securities must receive a prospectus no later than by receipt of the purchase confirmation. However, any solicitation must be preceded or accompanied by a prospectus.

All of the following statements regarding variable annuities are true except A) variable annuities offer the investor protection against capital loss. B) variable annuities are classified as insurance products. C) variable annuities may only be sold by registered representatives. D) insurance companies keep variable annuity funds in separate accounts from other insurance products.

A) A variable annuity is both an insurance and a securities product. An annuitant assumes the investment risk of a variable annuity and is not protected by the insurance company from capital losses.

If 1 OEX 375 call is purchased at 3.25 and exercised when the S&P 100 closes at 381, the writer delivers which of the following to the holder? A) $600 cash B) $381 in securities C) $325 cash D) $600 in stocks

A) Index options settle in cash. Physical delivery does not occur. The call buyer receives cash equal to the difference between the strike price and the index closing value on the day the option is exercised.

On February 13, your customer buys an 8% Treasury bond maturing in 2019 for settlement on February 14. If the bonds pay interest on January 1 and July 1, how many days of accrued interest are added to the buyer's price? A) 44 B) 43 C) 45 D) 14

A) Accrued interest for government bonds is figured on an actual-days-elapsed basis. The number of days begins with the previous coupon date and continues up to, but not including, the settlement date. The bonds pay interest on January 1. There are 31 days of accrued interest for January. The bonds settle February 14. There are 13 days of accrued interest for February. Do not count the settlement date (31 + 13 = 44 days).

A corporate bond pays interest on a J/J 15 schedule. An investor purchasing these bonds on Friday, April 17, would pay accrued interest for A) 96 days. B) 92 days. C) 95 days. D) 91 days.

A) Accrued interest on a corporate (or municipal) bond is based on each month containing 30 days. As with all bonds, the accrued interest is paid up to, but not including the settlement date. A trade made on Friday settles the following Tuesday (T+2), April 21. That means 3 months at 30 days each (90 days) plus 6 additional days (we don't count the settlement date) for a total of 96 days. One way to set this up is: 4/21 - 1/15 = 3 months and 6 days = 90 + 6.

A New York Stock Exchange designated market maker has all of the following responsibilities except A) preventing the stock price from falling. B) acting as broker for orders placed on the book. C) providing liquidity. D) handling odd-lot market orders.

A) Although designated market makers buy and sell as principals to maintain a fair and orderly market and provide liquidity, they are not obligated nor do they have a responsibility to prevent a decline in price.

When the SEC rules that an offering has become effective, the SEC has A) cleared the offering for sale. B) verified the accuracy of the statements in the registration statement. C) not verified the accuracy of each statement in the registration statement but has approved of the offering. D) approved the offering for registration.

A) An offering becomes effective when it is released by the SEC for sale. The SEC does not approve or disapprove of new offerings; it releases them for sale after determining that enough information is available for public investors to make sound investment decisions.

A fundamental analyst is reviewing GEMCO's financial statements. The company has a current ratio of 4:1, a price-to-earnings (P/E) ratio of 12:1, $10 million in 5% debentures, and net income after preferred dividends of $4 million. If the current market price of GEMCO stock is $60 and the company pays dividends at a rate of $0.75 quarterly, the dividend payout ratio is A) 60%. B) 20%. C) 5%. D) 40%.

A) As with many computational problems, there is some unnecessary information given. The current ratio is irrelevant, and so is the information on the debentures. What is needed is the amount available to pay the common so that we can compare that to the amount actually paid. We see that $0.75 in quarterly dividends are paid. That is equal to $3 per year. The next key is determining the earnings. With a market price of $60 per share and a price-to-earnings ratio of 12:1, the earning per share must be $5. The dividend payout ratio should be thought of as "dividends paid out of earnings made." The dividends paid are $3; the earnings made are $5. That is a 3 to 5 ratio, or, as usually expressed in percentage form, 60%.

Which of the following is least likely to be part of an equipment leasing partnership? A) Oil well casing and piping B) Computers C) Aircraft D) Railroad cars

A) Casing and piping are materials used in oil and gas well-drilling programs.

A New York Stock Exchange designated market maker is employed by A) a member of the exchange. B) the NYSE. C) the SEC. D) the OCC.

A) Designated market makers are employed by firms which must be exchange members.

Excess margin securities are defined as securities in excess of A) 140% of the customer's debit balance. B) the minimum maintenance margin requirements. C) 70% of the customer's debit balance. D) the customer's debit balance.

A) Excess margin securities are securities in excess of 140% of the customer's debit balance. Margin securities (140% of the debit balance) are at a bank collateralizing the customer's debit. For example, if a customer purchases $20,000 of stock, the customer will put up $10,000 and borrow $10,000. The member will take $14,000 of the stock to a bank to collateralize the $10,000 debit. The balance ($6,000) of the stock must be placed in segregation (excess margin securities).

Which of the following order types is permitted in Nasdaq markets but not in NYSE equity markets? A) Fill-or-kill (FOK) B) Limit C) Market D) Immediate-or-cancel (IOC)

A) FOK and all-or-none orders may no longer be entered in the NYSE equity market but are still accepted in both the bond market and Nasdaq.

A person legally responsible for the handling of the financial assets of another, such as an executor or guardian, is usually called A) a fiduciary. B) a trustee. C) a custodian. D) an investment adviser.

A) Fiduciary is the term that describes the legal position of trustees, custodians, and most investment advisers. This is a case where you choose the most complete response.

All of the following characteristics regarding industrial development bonds (IDBs) are true except A) the bonds are normally backed by the full faith and credit of the municipality. B) the bonds are issued by municipalities or other governmental units. C) the funds are used to construct a facility for a private corporation. D) funds from the lease are used to pay the principal and interest on the bonds. Explanation

A) IDBs are issued by a municipality, and the proceeds are used to construct facilities or purchase equipment for a private corporation. The corporation leases the facilities or equipment, and funds from the lease are used to repay investors. In addition to a first mortgage on the property, IDBs are backed by the full faith and credit of the corporation (not the municipality).

In a functional allocation oil and gas program, which of the following statements are true? The general partner picks up all tangible drilling costs. The general partner picks up all intangible drilling costs. The limited partners pick up all tangible drilling costs. The limited partners pick up all intangible drilling costs. A) I and IV B) III and IV C) I and II D) II and III

A) In a functional allocation program, the general partner picks up all tangible drilling costs, while the limited partners pick up all intangible drilling costs. As intangible drilling costs are deductible as they are incurred, this type of program benefits the limited partners. Tangible drilling costs, however, are deductible pro rata over the estimated life of the well.

An investor purchases five Mount Vernon Port Authority J & J 1 bonds in a regular way transaction on Wednesday, October 18. How many days of accrued interest are added to the bond's price? A) 109 B) 114 C) 108 D) 110

A) Interest accrues on municipal bonds on a 360-day-year basis, with all months having 30 days. This bond pays interest on January and July 1 (J & J 1). Therefore, July, August, and September each have 30 days of accrued interest, and October has 19 days of accrued interest; this totals 109 days. Settlement date is Friday, October 20.

Which of the following statements regarding Sallie Mae debentures are true? A) Interest is tax exempt at the state and local levels. B) Interest is paid monthly. C) They are backed by the taxing power of the U.S. government. D) Sallie Mae securities finance building public schools across the country.

A) Interest on nonmortgage-backed government securities is taxable at the federal level and exempt from state and local taxation. As a general rule, debentures pay interest every six months. Sallie Mae is not backed by the taxing power of the U.S. government, and money is used for student loans for higher education.

The interest that municipal securities pay is A) federally tax exempt. B) not taxed at the state, local, or federal levels. C) fully taxed. D) exempt from both state and local taxation.

A) Interest paid on securities issued by municipalities is generally exempt from taxation at the federal level. It may also be exempt from state and local taxation if the purchaser resides in the issuing state.

An investor purchased a single premium deferred variable annuity 20 years ago. The premium deposit was $50,000. The account is now worth $200,000 and the investor is still working. When does the investor have to begin taking required minimum distributions? A) Never with a nonqualified annuity B) At age 59½ C) At age 72 or when no longer working, whichever is later D) At age 72

A) On the exam, unless stated to the contrary, every annuity is nonqualified. One of the benefits of nonqualified annuities is that there is no age at withdrawals must commence. In general, earnings withdrawn prior to age 59½ are subject to the additional 10% penalty on top of tax at ordinary rates.

Regulation T permits borrowing money for the purchase of each of the following except A) listed options with expirations of less than nine months. B) listed warrants. C) listed stocks and bonds. D) unlisted stocks and bonds.

A) Options with expirations of less than nine months must be fully paid without exception. With some exceptions, warrants, stocks, and bonds may be purchased on margin.

As interest rates fall, prices of straight preferred stock will A) rise. B) fall. C) remain unaffected. D) become volatile.

A) Preferred stock is interest rate sensitive. As rates fall, prices of preferred stocks tend to rise, and vice versa.

An investor desiring a limited partnership investment with capital gains potential would most likely select one investing in A) raw land. B) shopping centers. C) oil and gas. D) equipment leasing.

A) Raw land Historically, raw land has been a source of appreciation. Shopping centers might appreciate in value but are oriented more toward current income. Equipment leasing offers income, and the asset ultimately depreciates. Oil and gas provide income as well, and the asset ultimately depletes.

An investor in an equipment-leasing direct participation program (DPP) using straight-line depreciation would probably not be concerned about A) the likelihood of recapture. B) liquidity risk. C) legislative risk. D) the quality of the management.

A) Recapture of deductions is a concern when accelerated, but not when straight-line depreciation is used. In any business, there is always concern about the quality of the management. By and large, DPPs are not liquid investments, so an investor needing a quick sale may have problems. The nature of DPPs tends to make them more sensitive to legislative risk than most other securities.

The child of one of your recently deceased clients comes to your office with several properly signed stock certificates inherited from a parent. The child does not have an account and wishes to sell the securities. An account is opened for the purpose of the liquidation. Regulation S-P would refer to this child as A) a consumer. B) a beneficiary. C) a covered person. D) a customer.

A) Regulation S-P makes a distinction between consumers and customers. That is important because it makes a difference when it comes to annual reporting. A consumer is basically a "one-shot" client, as in this case. After the liquidation, this account will be closed and you probably won't ever hear from the child again. A customer has an ongoing relationship and requires annual privacy notices—the consumer does not.

Gentry is the chief operating officer (CFO) of RMBM, a NYSE-listed corporation. Gentry has an account at your firm, and five months ago, Gentry purchased 1,000 shares of RMBM common stock at $50 per share. The RMBM shares are now $125 per share, and Gentry exits the position at that price. Which of the following statements presents the view of the SEC? A) Gentry has violated the short-swing profits rule. B) Gentry has violated the holding period requirements of Rule 144. C) Gentry has violated the volume requirements of Rule 144. D) Gentry has done nothing wrong because the stock was purchased in the open market.

A) Section 16 of the Securities Exchange Act of 1934 contains the short-swing profits rule. This rule states that any insider of a publicly traded corporation (the CFO would certainly be included in the definition of insider or affiliate) is prohibited from profiting from any purchase or sale (or sale and purchase) of the company's equity securities within a period of less than six months. This rule authorizes the corporation to recover from such statutory insider any so-called "short swing" profits. The term used in industry circles is that the profit must be disgorged (given back). There is nothing illegal here—no fines or penalties. However, we investors might consider returning a $75,000 profit to be a penalty. This stock was purchased in the secondary market, so the Rule 144 holding period does not apply. Rule 144 permits affiliates (like Gentry) to sell up to 1% of the outstanding shares over a 90-day period. RMBM is listed on the NYSE, and 1,000 shares is certainly much less than 1% of the shares outstanding. You do not need to know the listing requirements, but listing on the NYSE requires a minimum of 1.1 million shares outstanding.

Which of the following securities can generate phantom income? A) TIPS bonds B) Treasury bonds C) Treasury notes D) Treasury bills

A) TIPS bonds adjust the principal value each six months based on the inflation rate. If the inflation rate is positive, the value increases. Those increases are reported as income each year even though the investor does not receive the appreciation until the bonds mature (or are sold).

An individual purchases a variable life insurance policy. Under federal law, the individual is entitled to a complete refund of all premiums paid if the request is made within A) 45 days from the execution of the application, or for 10 days from the time the owner receives the policy, whichever is longer. B) the first 24 months after the policy was delivered to the owner. C) the first 30 days after the policy was delivered to the owner. D) 10 days from the execution of the application, or for 45 days from the time the owner receives the policy, whichever is longer.

A) The Investment Company Act of 1940 specifies a free-look period for the purchaser of a variable life insurance policy. That period is the longer of 45 days after the execution of the application or 10 days after the actual policy is delivered to the owner. The 24 months is the minimum time limit for the exchange of the variable policy into another form of permanent insurance.

The KPL Corporation is considering having its stock listed on the New York Stock Exchange (NYSE). Who will make the final decision as to whether it will be listed? A) The NYSE B) The Board of Directors of KPL C) The SEC D) FINRA

A) The NYSE has certain requirements that a company must meet before its stock can be considered for listing. Because the NYSE sets the requirements, it must make the final decision.term-103

Within a firm commitment underwriting, which document details the responsibilities and liabilities of each firm? A) Agreement among underwriters B) Letter of intent C) Underwriting agreement D) Registration statement

A) The agreement among underwriters, also called the syndicate letter, is signed by representatives of all syndicate members and establishes a joint account to sell newly issued securities.

For an investor in the 37% federal income tax bracket, the tax-equivalent yield of a general obligation municipal bond with a coupon rate of 4.17% is A) 6.62%. B) 5.72%. C) 11.27%. D) 2.63%.

A) The computation for tax-equivalent yield is done by dividing the coupon rate by (100% - tax bracket). In this question, that is 4.17% ÷ (100% - 37%) = 4.17%÷ 63%. That equals 6.62%.

All of the following statements regarding industrial revenue bonds (IRBs) are true except A) the credit rating of the bonds is dependent on the credit rating of the municipality. B) interest is paid from rental payments received from corporations that have leased the property or equipment from the municipality. C) they can be issued by municipalities to build facilities that will be owned by the municipality but leased to a local corporation. D) they can be issued by municipalities to provide local industries with funds for expansion.

A) The debt service for IRBs is derived from the lease payments made by the leasing corporation to the issuing municipality. Therefore, the credit rating of the bonds is dependent on the credit worthiness of the leasing corporation, not the issuing municipality.

One of your customers has recently celebrated a 58th birthday. The investor began a regular investment program into shares of the KAPCO Growth Fund over twenty years ago. The account is showing a substantial gain. Because retirement is getting closer, you suggest using the exchange privilege offered by the KAPCO fund group. Your recommendation is to place half of the holdings into the KAPCO Balanced Fund. Following this recommendation would result in A) a taxable transaction for those shares exchanged. B) a taxable transaction for those shares exchanged plus a 10% tax penalty for early withdrawal. C) tax deferral of any gains because the investor has not received any proceeds. D) tax deferral of any gains because the money is still in the KAPCO fund group.

A) The exchange or conversion privilege allows the investor to exchange shares of one fund in a family of funds for another at net asset value. The benefit is the saving of sales charge. The IRS treats this exchange as the sale of one security and the purchase of another. Therefore, any gains will be subject to tax. There is no 10% tax penalty. That applies only when there has been deferral of earnings, such as in an IRA.

An investor opens the following options position: Long 1 RAV Mar 50 put @5¾; short 1 RAV Mar 45 put @3. What is the investor's maximum gain, maximum loss, and breakeven point? A) Maximum gain is $225; maximum loss is $275; breakeven point is $47.25. B) Maximum gain is $275; maximum loss is $225; breakeven point is $47.25. C) Maximum gain is $275; maximum loss is $225; breakeven point is $47.75. D) Maximum gain is $225; maximum loss is $275; breakeven point is $47.75.

A) The first step is to identify the position. This is a debit put spread. It is a debit spread because the option purchased cost more than the one sold. The debit of $275 is the most the investor can lose. This is a bear put spread. We know that because the investor purchased the option with the higher strike price and sold the one with the lower strike price. The goal is for the stock's price to decline to the point where both options are exercised. For example, if the market price of RAV should fall below 45, the owner of the 45 put will exercise, causing the seller to purchase the stock for $4,500. The seller can then exercise the long 50 put and deliver the stock purchased at 45 for 50. That is a profit of $500 less the cost of the options (the debit of $275). The breakeven point follows the put-down rule. Subtract the net premium (the $2.75 debit) from the higher strike price resulting in a breakeven point at $47.25.

Question #113 of 125 Question ID: 1283065 An investor opens the following positions: Sell short 100 shares of BAF @61; short 1 BAF Sep 60 put @3¼. What is the customer's maximum gain, maximum loss, and breakeven point? A) Maximum gain is $425; maximum loss is unlimited; breakeven point is $64.25. B) Maximum gain is $325; maximum loss is unlimited; breakeven point is $57.75. C) Maximum gain is $5,775; maximum loss is $425; breakeven point is $64.25. D) Maximum gain is $425; maximum loss is $5,775; breakeven point is $57.75.

A) The first step is to identify the position. This is a short sale of stock and a sale of a put option. The sale of the put provides some income and offers protection only to the extent of the premium. Short sellers want the stock's price to decline. They lose when it rises. The investor has received $6,425 ($6,100 from the sale of the stock and $325 from the sale of the option). That makes the breakeven point $64.25 per share. Once the price of the BAF stock goes above that, the investor loses money. Because there is no limit as to how high the stock's price can go, the maximum loss is unlimited. If, on the other hand, the stock's price declines into the 50s or lower, the owner of the 60 put will exercise and our investor will pay $6,000 to purchase the stock. That stock will be used to cover the short sale. That means the investor sold the stock (short) at $61 and bought it back at $60 for a gain of $100. At that point, the investor's profit is the $300 from the premium on the sale of the put plus the $100 gain (the difference between 61 and 60). That is why the maximum gain is $425. Why doesn't the breakeven follow the "put-down" rule? That rule applies when the only positions are options. Once there is a long or short stock position along with an option position, it is the stock controlling the breakeven.

If an investor who has owned FLB stock for two years buys 1 FLB Oct put, this will A) have no effect on the holding period. B) end the holding period and cause any loss to be long term and any gain to be short term. C) end the holding period and cause any gain or loss to be short term. D) end the holding period and cause any gain or loss to be long term.

A) The investor already held the stock long term when the put was acquired, so there is no effect on the holding period.

A customer has a nonqualified variable annuity. Once the contract is annuitized, monthly payments to the customer are A) partially a tax-free return of capital and partially taxable. B) 100% tax deferred. C) 100% tax free. D) 100% taxable.

A) The investor has already paid tax on the contributions, but the earnings have grown tax deferred. When the annuitization option is selected, each payment represents both capital and earnings. The money paid in will be returned tax free, but the earnings portion will be taxed as ordinary income.

The locate requirement of Regulation SHO for short sales does not apply to A) nonconvertible bonds traded on the NYSE. B) American depositary receipts traded on the Nasdaq Stock Market. C) over-the-counter equity securities. D) preferred stock traded on the NYSE.

A) The locate requirement is applicable to all short sales of equity securities. It is unlikely to be tested, but, just in case, for purposes of this rule, convertible bonds are considered equity securities.

A convertible preferred stock with a par value of $100 is currently trading at $125 per share. The conversion ratio is 5:1. If the common stock is trading at $30 per share, what must the preferred stock's price be to be at parity? A) $150 B) $130 C) $103 D) $70

A) The math is 5 × $30 = $150. The logic is, you can convert the preferred stock into five shares of the common. If the common is trading at $30 per share, to be equal, the preferred stock must be selling for five times that price.

A client with an options account contacts the registered representative handling the account with instructions to open the following spread: Buy 1 ABC 100 call and Sell 1 ABC 105 call at a 5-point debit. Under FINRA rules, this order A) should be refused. B) should be turned in immediately. C) will be executed at the next available trade meeting the 5-point limit. D) is for a bull call spread.

A) The order should be refused because it is impossible for it to be profitable. This is a bull call spread (but that is not the correct answer here because it has nothing to do with FINRA rules) and will become profitable when the spread widens. With strike prices of 100 and 105, it can never widen more than 5 points. If the client paid 5 points for the spread, once commissions are factored in, the client must lose money and certainly cannot profit. FINRA looks at this as an uneconomic position, and the firm should refuse to take the order.

When a customer instructs a registered representative to transfer and ship, the representative instructs the margin department to transfer ownership into A) the customer's name and deliver the securities to the customer. B) the customer's name and deliver the securities to the customer's bank for safekeeping. C) the brokerage firm's name and deliver the securities to the customer. D) the brokerage firm's name and deliver the securities to the brokerage firm's commercial bank for safekeeping.

A) The term transfer and ship means to transfer the securities into the name of the customer and ship (deliver) the securities to the customer. To hold in street name would require the securities to be transferred into the name of the broker-dealer and held for safekeeping.

A producer of fine French wines has just signed a contract to export $10 million of wine to a distributor in the United States. Using listed foreign currency options, this producer would have the best protection against currency risk by A) taking a long position in euro calls. B) taking a long position in U.S. dollar puts. C) taking a long position in euro puts. D) taking a long position in U.S. dollar calls.

A) There are no listed options in the U.S. dollar. That reduces your choice to a long euro put or call. Because the contract will be paid for in dollars, the producer is concerned that the dollar will fall against the euro. Or, stated another way, the concern is that the euro will rise against the dollar so that the $10 million will not buy as many euros as on the day the contract was signed. When one is afraid the price of asset will rise, such as those who take a short position in a stock, the best protection is buying a call. The easiest way to remember this is through the acronym, IPEC - Importers buy Puts and Exporters buy Calls. This is used when the party involved in the question is in a foreign country. Because we are dealing with a French exporter, buying calls on the local currency offers the best protection.

According to the Investment Company Act of 1940, a diversified mutual fund may hold, at most, what percentage of a corporation's voting securities? A) 100% B) 5% C) 75% D) 50%

A) To be considered a diversified investment company, 75% of the fund's assets must be diversified, such that the mutual fund owns no more than 10% of a target company's voting securities. Additionally, within that 75% of assets, no diversified investment company may invest more than 5% of its portfolio in a single company's securities. However, there are no restrictions on the other 25%. That can all be in one stock, making 30% of the fund's assets in one company. Those assets can theoretically buy all of the outstanding voting shares of a company and control 100%.

An investor owns 500 shares of JKL common stock. The investor's cost is $50 per share and the last quote on JKL was $60. One method the investor could use that would increase current income and offer some downside protection would be to A) sell five JKL $60 calls @4. B) buy five JKL $60 puts @4. C) buy a JKL $60 put @4. D) sell a JKL $60 call @4.

A) Understand that with 500 shares, the protection needs five options. Selling five calls @4 generates $2,000 of premium income credited to the account one day after the sale. In addition, the sale has effectively lowered the cost from $60 to $56 per share. That is the partial protection offered when writing a covered call. Full protection would come from buying five JKL 60 puts, but the question specifically mentions receiving income and buying an option costs money rather than receiving money. Please note that some questions, like this one, do require you to consider the number of shares or options. Others, such as breakeven points, ignore the number of contracts.

Which of the following statements regarding a negotiated underwriting is not true? A) All states require that general obligation bonds (GO) must be underwritten using a negotiated process. B) The investment banker consults with the issuer to establish a price for the issue. C) Either municipal general obligation or revenue bonds can be underwritten on a negotiated basis. D) The municipality appoints the underwriter.

A) When a municipality appoints an underwriter, the bond issue is a negotiated underwriting. The price must be satisfactory to the issuer and still allow the underwriters to sell the bonds at a profit. There is no requirement that either municipal GO or revenue issues be underwritten as either negotiated or competitive bid. Each may be underwritten using either underwriting process.

Sagacious Publishing Company (SPC) has issued a $25 par 4% preferred stock. If current market interest rates should rise, it is probable that A) the market price of this stock will decline. B) investors holding shares of this stock will vote to increase the dividends. C) SPC will increase the dividends to match market returns. D) SPC will lower the par value of the stock.

A)Because preferred stock is a fixed-income investment, its market price moves inversely to changes in interest rates. As interest rates in the market go up, the price of preferred stock goes down. The dividend rate and par value of a preferred stock is fixed at issuance and do not change. It is rare to find a preferred stock with voting powers. Even then, they would never be voting on changing the dividend rate on a preferred stock. That is one of the benefits of adjustable-rate preferred stock. Because the dividend rate adjusts in response to changes in the market interest rate, the price generally remains relatively stable. How do we know this is not adjustable-rate preferred? Because the question would have to state that fact.

The computation for accrued interest on corporate and municipal debt obligations is based on

Accrued interest on corporate and municipal bonds is computed on a 30-day month and a 360-day year.

Gifts exceeding $100 may be given by a registered representative to A) a customer of the representative. B) the registered representative's unregistered sales assistant. C) a treasurer for an issuer of municipal securities. D) a registered representative of another member firm.

B There are no restrictions on giving gifts to colleagues employed by the same firm—registered or not. Registration would be required to split commissions, but not for making a gift.

A legal contract—known as an indenture—between a bond issuer and a trustee appointed to represent the bondholders is required for A) corporate bond issues of $25 million or more sold interstate. B) corporate bond issues of $50 million or more sold interstate. C) government bond issues of any size sold to domestic (U.S.) investors. D) municipal bond issues of $100 million or more sold within one municipality.

B)

An issuer may engage in a primary offering A) once per year. B) as often as it chooses. C) once every 90 days. D) once.

B)

SEC rules require that customers be given a copy of the risk disclosure document before their first transaction in a penny stock. The member firm must receive a signed and dated acknowledgment from the customer that the document has been received. In addition to obtaining the client's signature, the SEC requires the firm to wait at least A) five business days after receiving the statement before executing the first trade. B) two business days after sending the statement before executing the first trade. C) five business days after sending the statement before executing the first trade. D) two business days after receiving the statement before executing the first trade.

B)

Which of the following quotes represents a municipal dollar bond quote? A) 8.20 - 8.00 B) 85½ C) $850 - $870 D) 0.085

B) Dollar bond quotes are based on a percentage of face amount (Par $1,000). Therefore, a quote of 85½ is 85.5% of $1,000, or $855.

GIN Corporation is offering shareholders the right to subscribe to a new issue of stock at $30 per share. The current market price of the GIN stock is $44 per share, and it takes 20 rights plus the subscription price to purchase one share. The theoretical value of a single right, prior to the ex-rights date, is approximately A) $1.50. B) $0.67. C) $0.73. D) $0.70.

B) 0.67 Because this is cum rights (before the ex-rights date), the formula is M ‒ S N + 1 Thus, $44 ‒ $30 = 14 ÷ 21 = $0.67 20 + 1 * always remember if the question asks for the value of the right before the ex date (cum rights ) it is the number of rights +1, if it asks for the value of a right on or after the ex date it is without 1

If LEAPS options positions are maintained for more than 12 months, which of the following statements are true? The LEAPS writer's gains are taxed as short-term gains. The LEAPS writer's gains are taxed as long-term gains. The LEAPS buyer's gains are taxed as short-term gains. The LEAPS buyer's gains are taxed as long-term gains. A) II and IV B) I and IV C) II and III D) I and III

B) 1 and 4 The LEAPS writer's premium is taxed as a short-term gain. The LEAPS buyer took a position for longer than 12 months, so any profits are considered long-term capital gains. The writer's gain is short term because by opening with a sale, a holding period is never established.

A J & J Treasury bond with a 5% coupon due July 1, 2019, is purchased in a cash transaction on February 24. What is the number of days of accrued interest? A) 53 B) 54 C) 55 D) 63

B) A bond begins accruing interest on the prior interest payment date (January 1) and accrues up to, but not including, the settlement date (February 24). Did you notice that this was a cash transaction? That means the settlement date is the same day as the trade (February 24). Normally, Treasury securities settle T+1. If this was a regular-way trade, the accrued interest would be 55 days because settelment would have been February 25. Be careful reading the question; it is easy to skip over critical information. Because accrued interest on government bonds is computed actual days, actual year, 31 days for January plus 23 days for February, it equals 54 days.

If a customer sells short 100 ABC at 40 and sells 1 ABC Jun 40 put at 4, what is the maximum loss? A) $400 B) Unlimited C) $3,600 D) $0

B) A short stock position subjects an investor to unlimited risk. Writing a put against a short stock position only protects the investor to the extent of the premium received.

A company's changing from straight line to accelerated depreciation will increase income in the early years. decrease income in the early years. increase income in the later years. decrease income in the later years. A) I and IV B) II and III C) II and IV D) I and III

B) Accelerated depreciation increases charged expenses during the early years of equipment life but decreases charged expenses during the later years.

The determination as to whether an over-the-counter stock is eligible for purchase on margin is made by A) the Federal Deposit Insurance Corporation. B) the Federal Reserve Board. C) the Securities and Exchange Commission. D) the Financial Industry Regulatory Authority.

B) All decisions regarding initial margin eligibility are the role of the Federal Reserve Board.

Features of an employee stock purchase program (ESPP) include all of the following except A) the purchase price is discounted. B) contributions are made with pre-tax dollars. C) participants can sell the stock at any time. D) contributions are a percentage of pre-tax income.

B) Although the contributions are based on pre-tax income, unlike a qualified plan, the contributions are made with after-tax dollars.

With bonds subject to a gross revenue pledge, the first priority will be to pay A) operation and maintenance. B) bond interest and principal. C) the first lien on the property. D) the sinking or surplus fund.

B) Bonds subject to a gross revenue pledge (gross lien revenue bonds) are backed by the gross revenues of the facility (meaning revenues before expenses). In this case, the first money disbursed is for payment of interest and principal. However, most revenue bonds only pledge net revenues to pay off revenue bonds. In the more common net revenue pledge, the first priority is operation and maintenance; the second priority is interest and principal.

On exercise of the option, the holder of a put will realize a profit if the price of the underlying stock A) exceeds the exercise price plus the premium paid. B) falls below the exercise price minus the premium paid. C) exceeds the exercise price. D) falls below the exercise price.

B) Breakeven for the buyer of a put is the strike price of the option minus the premium paid for the option.

All of the following will cover a short call except A) a long position in the underlying stock. B) cash equal to the aggregate exercise value. C) an escrow receipt for the stock. D) a long call with a lower strike price and later expiration.

B) Cash never covers a short call because the cost to purchase the stock in the market for delivery at the strike price is unknown. If assigned, the customer must sell (deliver) at the strike price.

Holders of common shares may generally vote on A) whether a cash dividend is to be declared. B) whether the company should issue additional preferred stock. C) whether an administrative assistant should be promoted to management. D) which member of the board of directors should be chairman.

B) Common shareholders must vote to approve the issuance of additional preferred stock because additional preferred shares dilute the common shares' residual assets under a liquidation. Common shareholders do not vote to declare dividends. Board members select the chairman of the board. Shareholders do not get involved in the daily operational activity of the corporation.

Which of the following statements is true regarding dividend payments on common stock? A) Dividends on common stock are paid at the discretion of the board of directors and are paid as a stated percentage of the corporation's net income. B) Dividends on common stock are paid at the discretion of the board of directors and may be paid even where there are no earnings. C) Dividends on common stock are paid at the discretion of the board of directors and may be paid ahead of preferred stock when necessary to allow the company to remain listed on the exchange. D) Dividends on common stock are paid at the discretion of the board of directors and can be paid only when there are sufficient earnings.

B) Dividends on common stock are paid at the discretion of the corporation's board of directors. Although each stockholder receives an amount proportionate to their holdings, the dividend can be any proportion of the company's earnings. In fact, a corporation can pay a dividend even when there are no earnings. However, no dividend on common stock can ever be paid before payment of the dividends due on preferred stock.

Which of the following is considered a double-barreled bond? A) Dome stadium bonds with provisions for emergency ceiling support B) Bridge authority revenue bonds guaranteed by the full faith and credit of a city C) Build America Bonds D) Moral obligation bonds

B) Double-barreled bonds are backed not only by a specified source of revenues, but also by the full faith and credit of a municipal issuer with authority to levy taxes. Double-barreled bonds are sometimes classified in the broader category of general obligation bonds. The additional backing of PHAs is the full faith and credit of the U.S. government—not the issuer. The additional backing of moral obligation bonds are legislative appropriations, which are not mandatory.

Which of the following will halt trading in listed options when there is a trading halt in the underlying stock? A) The Options Clearing Corporation B) The options exchange on which the option is listed C) The Securities and Exchange Commission D) The exchange on which the stock is listed

B) If trading is halted in any stock on which options trade, trading in those options is also halted by the Chicago Board Options Exchange.

Intangible drilling costs would include all of the following except A) land surveys. B) casing. C) fuel. D) wages.

B) Intangible drilling costs are those associated with drilling a well, but do not include the cost of capital equipment (e.g., pumps, casing). They include wages, fuel, repairs, hauling, supplies, surveys, tests, and drilling mud, and they are incidental to and necessary for the drilling activity.

Your investor decides that she would like to open an options account. Which of the following is your responsibility as her registered representative? A) Assure that an options agreement has been signed before the first trade taking place B) Review with the client the risks involved when trading options and determine what types of options trading are appropriate for this client before the first options trade C) Obtain approval from the registered options principal to open the account no later than one business day after the first options trade D) Provide an options disclosure document no later than 15 days after the first trade

B) It is imperative that suitability and risk be addressed with the client before allowing option trading to take place.

When discussing mutual funds with a customer, which of the following statements is not prohibited? A) The income yield of the fund consists of both dividends and capital gains. B) Buy shares of different funds in the same fund family, and you may qualify for a breakpoint on the total purchase. C) Buy the shares on record date to receive the dividend. D) Get a few friends to join with you to form an investment club, and you may qualify for a breakpoint.

B) Most funds provide a combination privilege, allowing investors to aggregate purchases made in different funds in the same family to qualify for a breakpoint. The income yield of a mutual fund includes dividends only. A group of friends is not eligible for a breakpoint. (Investment clubs are not eligible.) Selling dividends is a prohibited practice because of the immediate tax liability incurred with the dividend and the share price adjustment that results after the dividend distribution.

All of the following securities trade in the over-the-counter (OTC) market except A) government and agency securities. B) open-end investment companies. C) Nasdaq securities. D) American depositary receipts.

B) Municipal bonds, government and agency securities, and corporate securities (listed and unlisted) all trade in the OTC market. Foreign securities trade in the United States if the companies comply with SEC registration and disclosure requirements. Mutual fund shares (open-end companies) do not trade.

The terminology guaranteed full faith and credit is most applicable to A) interest and principal on a corporate bond. B) interest and principal on a U.S. government-issued bond. C) interest only on a U.S. government-issued bond. D) interest and principal on a municipal revenue bond.

B) Of the choices given, the terminology would be most applicable to both interest and principal on a U.S. government bond. Remember that the U.S. government's guarantee is backed by their authority to tax and print money. While corporate bonds can be backed by the issuer's full faith and credit, the guarantee is only as good as the corporation's ability to pay. Municipal revenue bonds are backed by the expected revenue generated from the project being financed.

The primary purpose for creating ERISA was to A) promote a retirement fund for government employees. B) protect employees from the mishandling of retirement funds by corporations and unions. C) provide all employees, both government and nongovernment, with an additional source of retirement income in the event that the Social Security system defaults. D) establish a means for self-employed persons to provide for their own retirement.

B) Prevent abuse and misuse of pension funds guidelines apply to private sector (corporations and certain union plans - not public plans like those for the government workers ERISA was created to protect the retirement funds of union members and employees of large corporations. ERISA guidelines state that all qualified retirement plans must be in writing, segregate funds from corporate or union assets, make prudent investments, report to participants annually, and not be discriminatory. All of these activities are audited under ERISA.

With the advent of the horseless carriage (a.k.a. the automobile), the Acme Buggy Whip Corporation's revenues fell to the point where it could no longer cover expenses. This led to an involuntary bankruptcy. The priority of payout was A) senior notes, preferred stock, common stock, general creditors. B) senior notes, general creditors, preferred stock, common stock. C) general creditors, senior notes, preferred stock, common stock. D) common stock, preferred stock, general creditors, senior notes.

B) Senior debt refers to obligations that have priority in the event of default. It parallels the use of senior when comparing preferred stock to common stock, the most junior of all securities.

Which of the following would least likely occur when a corporation engaged in a rights offering? A) The number of outstanding shares would increase. B) After successful completion of the offering, the market price would rise slightly. C) The corporation would use a standby underwriter. D) After successful completion of the offering, the market price would decline slightly.

B) Successful completion of a rights offering generally results in a slight decline in the market price of the stock. This is because the subscribers were able to purchase at a price below the current market. This would have a small dilutive effect, causing a slight reduction in the market price. The rights offering is of additional shares, so the number outstanding would increase. Most corporations use a standby underwriter who will buy any shares that were not exercised.

A moral obligation bond is one where A) the U.S. government is empowered, but not obligated to appropriate funds to prevent the bond from going into default. B) the governing legislature is empowered, but not obligated to appropriate funds to prevent the bond from going into default. C) the bondholders are empowered, but not obligated to appropriate funds to prevent the bond from going into default. D) the board of directors of the entity is empowered, but not obligated to appropriate funds to prevent the bond from going into default.

B) The MSRB defines a moral obligation bond as: "A bond that, in addition to its primary source of security, is also secured by non-binding covenant that any amount necessary to make up any deficiency in debt service will be included in the budget recommendation made to the governing body, which may appropriate funds to make up the shortfall. The governing body, however, is not legally obligated to make such an appropriation."

The Municipal Securities Rulemaking Board (MSRB) is authorized to adopt rules concerning all of the following except A) the form and content of price quotations. B) the information to be provided by municipal issuers. C) the sale of new issues to related portfolios. D) the regulation of municipal securities advertising.

B) The MSRB does not regulate issuers. Rather, it regulates the underwriting of municipal securities and subsequent secondary market trading. Disclosure requirements for issuers are mandated by the SEC.

All of the following are required by limited partnerships except A) a subscription agreement. B) SEC approval. C) a partnership agreement. D) a certificate of limited partnership.

B) The SEC does not approve limited partnerships or any other securities. In public offerings of limited partnerships (as opposed to private placements), federal registration and a prospectus are required.

A 60-year-old customer who wishes an investment that can provide for retirement needs while adjusting for changes as aging takes place would probably find which of the following investments most suitable? A) An IRA B) A target date fund C) An asset allocation fund D) A long-term bond fund

B) The benefit of a target date fund is that, as the specified date gets closer, the portfolio allocation automatically shifts into more conservative investments. It could be compared to an asset allocation fund on autopilot. An IRA is not an investment vehicle and at age 60 is probably not the best time to get started. The long-term bond fund is going to suffer from inflation risk, as well as interest rate risk.

Question #27 of 125 Question ID: 1309808 Class A shares of the JILCO Fund are offered to the public at net asset value per share plus a sales charge of 4.25%. If the next computed net asset value per share is $39.39, an investment of $10,000 will acquire A) 243.546 shares. B) 243.072 shares. C) 253.871 shares. D) 265.816 shares.

B) The first step is computing the public offering price (POP). When the NAV and sales charge percentage are given, the computation is: NAV divided by (100% - sales charge %). The numbers are $39.39 ÷ 95.75% which equals $41.14 per share. Then, divide the $10,000 purchase by the POP of $41.14 to arrive at 243.72 shares. The key point to remember is that the sales charge is always a percentage of the POP, not the NAV.

Which of the following types of retirement plans would be most beneficial to a young employee of a corporation? A) Profit-sharing plan B) Defined contribution pension plan C) Keogh plan D) Defined benefit pension plan

B) The most beneficial corporate pension plan for a younger employee would be the defined contribution plan. The employee has many years in the workforce, so the investments made with the defined contributions will have a maximum amount of time to grow.

A confirmation sent to a customer must include all of the following except A) whether the member acted as an agent or principal. B) the name of the registered representative handling the account. C) markup or markdown if the member acted as a principal in a Nasdaq security. D) the amount of any commission.

B) There is no requirement to provide identifying information for the registered representative. A customer confirmation must disclose the amount of markup for a principal transaction in a Nasdaq security, whether the member acted in an agency or principal capacity, and the amount of commission if the member acted as an agent.

The MSRB defines an associated person of a broker-dealer who is primarily engaged in municipal securities activities other than retail sales to individuals as A)a municipal securities professional (MSP). B)a municipal financial professional (MFP). C)a municipal securities registered principal (Series 53). D)a municipal securities registered representative (Series 52).

B) These MFPs are involved with functions other that retail sales of municipal bonds. The definition is an outgrowth of the "pay to play" rule because these are the individuals covered by that rule.

The City of Podunk has an outstanding 25-year maturity issue that is callable in seven years. It has prerefunded the issue and established an escrow account containing the proper government securities with face amounts and maturities approximating the call provisions of the original issue. In quoting the original issue, which of the following must be used? A) Yield to maturity B) Yield to call C) The lower of the yield to call or the yield to maturity D) Current yield

B) When a bond issue is prerefunded, the issuer is going to redeem the bond on the first call date. The yield must be quoted to call.

Lambda Corporation has received a donation of 100,000 shares of its common stock from the spouse of the deceased founder of the company. This would appear on the company's books as A) unissued stock. B) treasury stock. C) authorized, but unissued stock. D) reacquired stock.

B) When a corporation reacquires shares of outstanding stock, whether through open market purchase or, as in this case, donation, the stock is treasury stock.

In which of the following situations may a broker-dealer enter an order for a customer to sell a stock long? The broker-dealer has reason to believe that the customer owns the stock and will deliver it promptly. The security is carried in the customer's account at the broker-dealer. The customer owns a bond convertible into the stock and has issued conversion instructions. The customer owns a call option on the stock and has exercised the call. A) I and III B) I, II, III, and IV C) I and II D) II and IV

B) all of them. To note that a sell order is long, a broker-dealer must either have the stock in its possession or have reasonable assurance that the customer owns the stock and will deliver it promptly. Regarding a convertible bond or a call option, to be considered long the stock a customer must have tendered the bond for conversion or have given notice to exercise the call.

All of the following pieces of information must be obtained from new individual customers except A) residence address. B) educational background. C) Social Security number. D) date of birth.

B)A customer's educational background is not required to open a new account. In the case of an account opened in a business's name, the business address and tax identification number are required.

The document attesting to the formation of a limited partnership, filed with designated authorities, is called A) the subscription agreement. B) the registration statement. C) the certificate of limited partnership. D) the offering memorandum.

C ) The Uniform Limited Partnership Act requires that two or more persons sign and swear to a certificate of limited partnership. It is filed with the state and is a public document available for review.

A married couple who files jointly has a $5,000 long-term capital loss with no offsetting capital gains. Regarding the tax treatment of this loss, all of the following statements are true except A) they can carry forward $2,000 to future years. B) capital losses can be used to offset capital gains only. C) capital losses can be deducted dollar for dollar against capital gains. D) the maximum they can deduct this year is $3,000.

C)

A mutual fund can use the term, "no-load" as long as A) any 12b-1 charge does not exceed 1.00%. B) any 12b-1 charge does not exceed .75%. C) any 12b-1 charge does not exceed .25%. D) there is no 12b-1 charge.

C)

An investor, long 100 shares of XYZ at $23.50, writes 1 XYZ May 25 call at 2. At expiration, if XYZ is trading at $20, the investor has A) an unrealized loss of $50. B) an unrealized gain of $50. C) an unrealized loss of $150. D) an unrealized gain of $150.

C)

A head and shoulders bottom formation is an indication of A) a bullish market. B) a bearish market. C) the reversal of a downtrend. D) the reversal of an upward trend.

C) A head and shoulders bottom formation is also known as an inverted head and shoulders formation. It is that part of a graph in which a downtrend has reversed to become an uptrend. It is not, however, an indicator of the bullishness or bearishness of the market as a whole. It is an indication only of the direction of a trend, which may be either short or long in duration.

Which type of risk is a mortgage-backed security most likely to experience? A) Exchange rate risk B) Business or corporate risk C) Reinvestment rate risk D) Market risk

C) A mortgage-backed security, such as a collateralized mortgage obligation, is most likely to experience reinvestment rate risk. As mortgages are paid off early and refinanced in the event of declining interest rates, the interim cash flows received from the obligation must be reinvested in lower yielding securities. This is the practical effect of prepayment risk.

A gain on the sale of a long equity put option is A) ordinary income. B) always a long-term capital gain. C) always a short-term capital gain. D) a short- or long-term capital gain.

C) Any trading in options produces only short-term gains or losses; therefore, any gain on the sale of a long put option must always be a short-term capital gain. (If a question wishes you to consider LEAPS, the question will refer to them.)

If a customer writes 1 ABC Nov 60 put at 3.50, and the put is exercised when ABC is 57.50, the customer's cost basis in ABC stock is A) $57.50. B) $60.00. C) $56.50. D) $54.00.

C) At exercise, the premium of the contract affects the cost basis of the stock acquired. Because the premium of 3.50 was received when the put was written, the cost basis of the stock will be $60 per share less the premium, or 56.50.

If an investor buys 300 shares of FLB, and one month, later buys 1 FLB Jul 50 put, how does this affect the holding period on his stock? A) It ends the holding period on the put. B) It has no impact on the holding period for any of the shares owned by the investor. C) It erases the holding period on 100 shares. D) It erases the holding period on 300 shares. Explanation

C) Because the stock has not been held more than 12 months, the put purchase erases the holding period for any shares the put subsequently allows the holder to sell. Because the holder owns one put, this erases the holding period on 100 shares owned. The other 200 shares are unaffected.

All of the following statements regarding collateralized mortgage obligations (CMOs) are true except A) CMOs are a derivative security. B) interest payments are distributed pro rata when received. C) interest is paid semiannually. D) principal repayments are applied to earlier tranches first.

C) CMO holders are paid interest monthly. As payments are received from the underlying mortgages, interest is paid pro rata to all tranches, but principal repayments are paid to the first tranche until it is retired. Subsequent principal repayments are then applied to the second tranche until it is retired and so on. CMOs are a derivative security because the value of each tranche is derived from the timing of principal repayments to that tranche.

All of the following are suitable objectives for a covered call writer except A) providing downside protection for a long stock position. B) speculating that a stock will not rise in price. C) profiting from an increase in the price of stock. D) increasing return on a long stock position.

C) Covered call writers are not able to benefit from an increase in the price of the underlying stock. For example, you buy stock at $40 and write a 40 call. Now, the stock is 80. Isn't that great? Your long stock position has doubled. Not so fast. With the stock at 80, it is certain that the 40 call will be exercised. So no matter how high the stock goes, the covered writer can't benefit because the call will be exercised and the stock will be sold at the $40 strike price. Why sell covered calls? This strategy provides downside protection to the extent of the premium received, and it increases the rate of return on a long stock position (because of the premium collected).

A registered representative with discretionary authority requires customer authorization before purchasing A) noncallable zero-coupon bonds. B) junk bond funds. C) municipal bonds where a control relationship exists. D) noninvestment-grade bonds.

C) Even though the representative has discretionary authority to trade the account, the Municipal Securities Rulemaking Board requires that the representative receive customer permission before purchasing bonds where the firm has a control relationship with the issuer.

A married couple both hoping to retire within the next five to seven years have expressed having a low-risk tolerance regarding the stock market. They have a combined income of $350,000. Given this information, which of the following portfolio mixes would be most suitable? A) Treasury bills, common stock, options B) Treasury bills, corporate bonds, preferred stock C) Treasury notes, municipal bonds, GNMAs D) Direct participation programs, real estate investment trusts, preferred stock

C) In light of their low risk tolerance, U.S. government securities would certainly be suitable, and the time frame noted for retirement allows for middle term T-notes to be useful. Given their higher income level, tax-free municipal bonds could also have a place in the portfolio. Longer term GNMAs would accommodate monthly income, should that be desirable upon retirement. The remaining product suggestions are either illiquid (DPPs) or do not align with their risk aversion (common, preferred, options, and REITs).

When determining position limits for listed options contracts and LEAPS contracts on the same side of the market, which of the following statements is true? A) The contracts are added to increase the position limits. B) The contracts do not have position limits. C) The contracts must be aggregated. D) The contracts are considered separately. Explanation

C) LEAPS and listed options on the same side of the market, on the same underlying security, must be aggregated and remain within position limits.

The termmunicipal fund security refers to A) a municipal bond with a sinking fund. B) a mutual fund whose portfolio is exclusively municipal bonds. C) a Section 529 savings plan. D) an advance refunded municipal bond.

C) Section 529 plans, used primary for saving for college, are legally considered municipal fund securities.

Which of the following would be considered an equity security? A) An exchange-traded note B) A collateralized mortgage obligation C) A prior lien preferred stock D) An equity-linked note

C) Stock means equity. Prior lien means that this preferred has priority over other preferred stock the company has issued. The other three are alternative forms of debt financing. Do not fall into the equity-linked note trap; it is a debt security.

The 5% markup policy would apply to all of the following equity transactions except A) an agency trade done on an exchange. B) a proceeds transaction. C) a primary market transaction. D) a riskless principal transaction.

C) The 5% markup policy applies to secondary market transactions in nonexempt securities.

Variable annuities generally include an assumed interest rate. This is A) the rate used to illustrate the future growth prospects of the contract. B) the annual dividend rate that will be paid to contract holders. C) the annual rate of return required to maintain the level of annuity payments. D) the annual rate at which annuity payments will increase.

C) The assumed interest rate (AIR) is the rate the insurance company assumes the separate account will earn during the payout period. If the assumption is wrong, the monthly payments will be adjusted accordingly. If the separate account earns more than the AIR, the next month's payment is increased. If the separate account earns less than the AIR, the next month's payment is reduced. If the account earns the assumed rate, monthly payments will not change.

Which of the following would not be found in a municipal revenue bond resolution? A) Conditions of the maintenance covenant B) Reporting requirements regarding revenues collected C) Underwriting agreement D) Terms of the rate covenant

C) The bond resolution, which is also referred to as the bond contract, contains the requirement for the municipality to properly keep the facilities books, reporting requirements regarding revenues collected, conditions of the maintenance covenant, and terms of the rate covenant. The underwriting agreement is between the municipality and underwriters, and it spells out the terms agreed to for the underwriting of a new issue.

The call premium on a municipal bond trading above par is best described as the difference between A) the amortized premium and the annual interest. B) the market price and the call price. C) par and the call price. D) the market price and par.

C) The call premium represents the difference between the call price and par. The farther away a call date, the lower the call premium.

All of the following would be considered when evaluating a municipal revenue bond's creditworthiness except A) coverage ratio. B) management expense. C) collection ratio. D) competing facilities.

C) The collection ratio shows the percentage of property taxes that are collected. This would be relevant in evaluating general obligation bonds, which are backed by the taxing authority of the issuer. Revenue bonds, however, are backed by user fees, not taxes.

Which of the following statements regarding both traditional and Roth IRAs is true? A) Withdrawals at retirement are tax free. B) Distributions must begin in the year after the owner reaches age 72. C) Contribution limits are the same. D) Contributions are tax deductible.

C) The common factor for both traditional and Roth IRAs is that contribution limits are identical. A significant difference between the two is that Roth IRAs do not have RMDs.

Covered put writing is a strategy where an investor A) sells a put and sells a call on the same stock. B) sells a put on a stock that he owns. C) sells a put on a stock he has sold short. D) sells a put and buys a call on the same stock.

C) The customer sells the put to generate income. The short stock position provides the necessary cash should his short put be exercised, forcing him to buy the stock.

Which of the following is least likely to be a risk concern to an investor in an oil and gas DPP? A) Lack of liquidity B) Risk of an IRS audit C) Deductions for intangible drilling costs D) Legislative risk

C) The deductions for intangible drilling costs are a benefit rather than a risk. If the choice said excess intangible drilling costs, then the investor might be subject to the alternative minimum tax (AMT). On the exam, you can count on DPPs having liquidity risk. It is generally believed that tax returns showing ownership of a DPP have a greater audit risk. Because one of the features of a DPP is the tax treatment, a potential change to the tax laws by the Congress constitutes legislative risk.

A corporation's income statement reports net income of $10 million for the year. The company has one million shares of 4% $50 par value preferred stock and two million shares of common stock. If the corporation paid a quarterly dividend of $0.60 per share of common stock, A) the earnings per share was $5 per share. B) the retained earnings increased by $6.8 million. C) the dividend payout ratio was 60%. D) the current return on the preferred stock was 4%. Explanation

C) The dividend payout ratio is the percentage of the net income (after preferred stock dividends) paid out to the common shareholders. The net income is $10 million. The preferred dividend is $2 million (1 million shares at $2 per share). That leaves $8 million in earnings available to common. There are 2 million shares receiving an annual dividend of $2.40. That means $4.8 million of the $8 million available is paid, or a ratio of 60%. Or, the earnings per share is $4.00 ($8 million divided by 2 million shares) and $2.40 in dividends paid out of $4.00 earnings made is 60%. The preferred stock is paying a dividend of 4% of the par value, but that does not tell us the current yield. To know the current yield, we must know the current market price of the stock and the question does not supply that value.

Smith and Company, a FINRA member firm, is preparing to underwrite securities to be issued by KLC Corporation for a new business venture. For which of the following will Smith and Company be responsible? Filing the registration statement with the SEC and state regulatory bodies Providing advice on the type of security to be issued Distributing the security to the public Providing advice on how KLC can best use the funds raised A) II and IV B) I and IV C) II and III D) I and III

C) The issuer is ultimately responsible for filing registration statements with federal and state regulatory bodies and has already determined how the money will be used. The underwriter confines his activities and advice to the type and sale of the securities.

Whose Social Security number must open a custodial account? A) A custodian's, parent or guardian's, and minor's B) A parent or guardian's C) A minor's D) A custodian's

C) The minor's Social Security number is required because the account is fully owned by, and taxed to, the minor—not to the custodian or the parent.

Your broker-dealer acts as a prime broker for ABC Fund. In this arrangement, your broker-dealer is likely providing which of the following services? Execution of all transactions for the fund portfolio Clearing services Lending for trades done on margin Ensuring that all exchange trading rules are complied with A) I and IV B) I and III C) II and III D) II and IV

C) The prime broker would supply clearing services and lending services for a marginable transaction, as well as back-office support such as cash management, account statements, and transaction processing. Actual executions and abiding by all exchange rules when transactions occur is the responsibility of the executing broker-dealers.

The Securities Act of 1933 covers all of the following except A) liabilities for misleading filings. B) prospectus requirements. C) blue-sky laws. D) full and fair disclosure.

C) The purpose of the Securities Act of 1933 is to provide investors with full disclosure about a new securities issue. The act is federal in scope, whereas blue-sky laws refer to state securities regulations.

The underwriting agreement is signed by A) the selling group members and the syndicate members. B) the issuer and the SEC. C) the issuer and the managing underwriter. D) the managing underwriter and the syndicate members.

C) The underwriting manager represents all underwriting members, and on behalf of the underwriting group, signs the underwriting agreement with the issuer. The agreement among underwriters is the document signed by the managing underwriter and all syndicate members. The selling group members have no formal agreement to be signed with the underwriters.

Which of the following securities underlies a yield-based option? A) Debentures B) Revenue bonds C) Treasury securities D) Income bonds

C) Treasury Securities Yield-based interest rate options are based on the yields of Treasury bills, notes, and bonds.

If an investor keeps $100,000 invested in U.S. Treasury bills at all times during a 10-year period, she is subject to which of the following? Stable principal Unstable principal Stable interest Unstable interest A) II and III B) II and IV C) I and IV D) I and III

C) Treasury bills are purchased at a discount and mature at face value. This feature provides principal stability to investors who own them. The discount on bills is determined by current market interest rates and fluctuates accordingly.

PDQ Corporation has a 6.25% $100 par value convertible preferred stock (conversion ratio of 4) outstanding. The stock has an antidilution covenant. If PDQ declares a 10% stock dividend, the antidilution covenant will adjust A) the par to $90. B) the par to $110. C) the conversion price to approximately $22.73. D) the conversion price to approximately $27.50.

C) When a $100 par value preferred stock is convertible into four shares of common stock, the conversion price is $25 per share, ($100 ÷ 4 = $25). The antidilution covenant means the investors will have the same conversion rights after a stock split or stock dividend as they had before. After a 10% stock dividend, each share of preferred stock will be convertible into 4.4 shares (4 shares x 110% = 4.4). The par value of the preferred stock does not change. Divide that $100 par value by the new number of shares to get the new conversion price. It looks like this: $100 ÷ 4.4 = 22.73. Alternatively, you can divide the original conversion price of $25 by 110% arrive at the same answer.

Which of the following regarding yield-based (interest rate) debt options is true? A) Debt securities are delivered to the contract owner when exercised. B) Their strike prices reflect dollar amounts. C) They are European-style exercise. D) Calls are purchased by those who believe prices of debt securities are rising.

C) Yield-based debt options are European-style contracts, meaning that they can only be exercised on the last day of trading. All yield-based contracts, when exercised, are settled in cash. There is no delivery of debt instruments when these contracts are exercised. All strike prices reflect yield. (35 strike price represents 3.5% yield.) Yield-based options are a bet on future interest rates, not prices. Calls are bought by those who believe rates are going up (prices down) and puts by those who believe rates are going down (prices up).

You received a signed broker-to-broker transfer initiation form (TIF) from an established customer desiring to transfer a specifically designated part of his account to your firm, which is eligible to use the Automated Customer Account Transfer Service (ACATS). Your firm is obligated to submit the transfer instruction to the carrying member by establishing the instruction in the ACATS A) within 1 business day. B) within 2 business days. C) Immediately. D) within 3 business days.

C) immediately FINRA's Uniform Practice Code requires that the receiving member firms immediately forward the TIF to the firm currently carrying the account. A customer may transfer all or part of the securities held in the account.

A customer requests information on a variable life insurance policy and asks her registered representative to circle the important information in the prospectus and information he thinks will be of special interest to her. This is permitted A) without restriction. B) if approved by a principal. C) under no circumstances. D) if accompanied by an unmarked prospectus.

C) the prospectus is a legal document that may not be altered

Which of the following regarding T-bills are true? T-bills trade at a discount to par. T-bills have maturities of 1 to 10 years. Most T-bill issues are callable. T-bills are a direct obligation of the U.S. government. A) II and IV B) II and III C) I and IV D) I and III

C)T-bills trade at a discount to par, are six months or less to maturity, and are a direct obligation of the U.S. government. T-bills are also noncallable.

Which of the following is not a correct statement in respect to the at-risk provisions when investing in a direct participation program (DPP)? A) Deductions or losses are limited to the investors' invested capital plus their percentage of partnership liabilities for which they are personally liable. B) Losses disallowed by the at-risk provisions in any one year may be carried over to following taxable years. C) The at-risk provisions do not apply to oil and gas exploration programs. D) Qualified nonrecourse financing is excluded from tax basis except in the real estate programs.

C)The at-risk provisions (you can only deduct what you can lose) apply to all DPPs. Real estate has one unique feature in that nonrecourse financing is part of the investor's tax basis.

Which of the following statements regarding Treasury receipts is not true? A) Treasury securities held in trust collateralize the receipts. B) Treasury receipts are not backed by the faith and credit of the U.S. government. C) Interest income is taxed at maturity. D) Treasury receipts pay interest at maturity.

C)Unlike Treasury STRIPS, which are issued directly by the U.S. government, Treasury receipts are indirect obligations of the government. Treasury receipts are issued by investment bankers who buy Treasury securities, place them in trust at a bank, and sell separate receipts against the principal and interest payments. Like most zeroes, interest must be accreted and taxed annually even though it is not received until maturity. LO 7.a

Elisha purchased 100 shares of RMBN common stock on June 6, 2019, at $60 per share. On February 11, 2020, RMBN paid shareholders a 20% stock dividend. Elisha sells the shares received as the stock dividend on December 5, 2020, at $55 per share. What are the tax consequences of this trade? A) $100 short-term capital loss B) $100 short-term capital gain C) $100 long-term capital gain D) $100 short-term capital loss

C)When a stock dividend is paid, the cost basis of the shares is adjusted. In this case, Elisha now owns 120 shares and the total cost is still the original $6,000. That makes the adjusted cost basis per share $50 ($6,000 ÷ 120). With the new cost basis of $50 per share, when the sale of those 20 shares takes place at $55 per share, the result is a gain of $100 ($5 per share profit times 20 shares. Alternatively, $1,100 total proceeds [20 shares x $55 per shares] minus $1,000 cost basis [20 shares x $50 per share adjusted cost per share)). Even though these shares were acquired less than 12 months before the sale, their holding period is based on the original purchase date and that is clearly more than 12 months before the sale. That is why it is long term

An investor opens a long position in one XYZ Nov 140 put @7. Disregarding any commissions, if the option is exercised, on settlement date the investor A) receives $700. B) must pay $700. C) receives $14,000. D) must pay $14,000.

C)When an investor takes a long position in an option, it means that the investor has purchased the option. When a put is exercised, the holder must deliver the stock on settlement date. At that time, proceeds representing the strike price ($140) for 100 shares ($14,000) are received.

A customer buys 200 XYZ at 39 and writes 2 XYZ Feb 40 calls at 3. When the stock rises to 44, the customer is exercised for a gain of A) $400. B) $200. C) $1,600. D) $800.

D) 800The customer bought 200 shares at 39 and was forced to sell them at 40 for a $200 gain. In addition, the customer received $600 in premium income, so the overall gain is $800. Alternatively, the breakeven point for covered call writing is cost of shares purchased less premium received (39 − 3 = 36). As the customer is bullish, gain occurs above 36. However, for this customer, the stock can go no higher than 40 because she will be exercised (40 − 36 = 4 points × 200 shares = $800).

A mutual fund's expense ratio is found by dividing its expenses by its A) income. B) dividends. C) public offering price. D) average annual net assets.

D

What is a possible benefit of purchasing shares of a closed-end investment company in the secondary market? A) Redemption takes place on the day of the sale. B) Shares may be purchased and sold without any sales or redemption charge. C) Dividends are paid in cash rather than additional shares. D) Shares are frequently trading at a discount to the NAV.

D)

When a customer enters a sell order and is in possession of the certificates, a broker-dealer must determine all of the following except A) the location of the securities. B) whether the client can make delivery promptly. C) whether the transfer agent has accepted the securities. D) whether the securities are in deliverable form.

D)

Which of the following organizations determines which over-the-counter securities are eligible for purchase on margin? A) Securities and Exchange Commission B) Municipal Securities Rulemaking Board C) Financial Industry Regulatory Authority D) Federal Reserve Board

D)

Which of the following statements regarding the exercise of options contracts are true? The exercise of equity options settles the next business day. The exercise of equity options settles in two business days. The exercise of index options settles the next business day. The exercise of index options settles in two business days. A) I and III B) I and IV C) II and IV D) II and III

D) Listed equity options, if exercised, settle in two business days (regular way settlement for equities). Index options, if exercised, settle on the next business day—and in cash.

FINRA Rule 2310 defines a direct participation program as "a program which provides for flow-through tax consequences regardless of the structure of the legal entity or vehicle for distribution including, but not limited to, oil and gas programs, real estate programs, agricultural programs, cattle programs, condominium securities, Subchapter S corporate offerings and all other programs of a similar nature, regardless of the industry represented by the program, or any combination thereof." The rule places limits on the overall expenses and amount of broker-dealer compensation considered fair and reasonable. That limit is A) 10% of the gross proceeds. B) 2% of the gross proceeds. C) 5% of the gross proceeds. D) 15% of the gross proceeds.

D) 15% If the organization and offering expenses exceed 15% of the gross proceeds, FINRA considers that too high. The 10% limitation is on the amount of compensation received by a member firm for selling interests in the DPP. The 2% is the maximum charge in a DPP rollup if the firm wishes to solicit votes from the limited partners. The 5% is the FINRA markup policy and that does not apply to DPPs.

Which of the following statements about municipal brokers' brokers is not true? A) They do not maintain inventories. B) They do not perform retail trades with individual investors. C) They perform specialized trades for institutions. D) They perform trades on a principal basis only.

D) A broker's broker does not maintain an inventory of bonds. Therefore, they do not act as principals; they act as agents only in trades between dealers or institutions. They do not do retail business.

A taxpayer's most advantageous tax benefit is A) straight-line depreciation. B) a tax deduction. C) a depletion allowance. D) a tax credit.

D) A tax credit reduces a person's tax liability dollar for dollar. Deductions, depreciation, and depletion reduce taxable income.

The antifraud provisions of the Securities Exchange Act of 1934 apply to all of the following except A) Nasdaq- and exchange-listed securities. B) options. C) municipal bonds. D) commodities.

D) All securities are subject to the antifraud provisions of federal securities laws. It should be recognized that commodities like wheat or oil are not securities.

According to Municipal Securities Rulemaking Board (MSRB) rules, a municipal finance professional (MFP) is an associated person of a broker-dealer engaged in municipal securities representative activities other than retail sales. a registered representative engaged in the retail sale of municipal securities to individual investors. subject to the political contribution rules as outlined in MSRB Rule G-37. not subject to the political contribution rules as outlined in MSRB Rule G-37. A) II and III B) II and IV C) I and IV D) I and III

D) As defined by the MSRB, an MFP is an associated person of a broker-dealer who is primarily engaged in municipal securities representative activities other than retail sales to individuals, who solicits municipal securities business for the broker-dealer, or who is in the supervisory chain above MFPs as described. MFPs are subject to the political contribution reporting rules as outlined in MSRB Rule G-37. Though there are exceptions for de minimis contributions, exceeding the allowable contribution limits may trigger restrictions on engaging in municipal securities business by the broker-dealer for two years. LO 6.h

An investor opens the following positions: Buy 100 shares of RJN @46; buy 1 RJN Mar 45 put @2½. What is the customer's maximum gain, maximum loss, and breakeven point? A) Maximum gain is unlimited; maximum loss is $350; breakeven point is $42.50. B) Maximum gain is $350; maximum loss is $4,250; breakeven point is $42.50. C) Maximum gain is $4,250; maximum loss is $250; breakeven point is $48.50. D) Maximum gain is unlimited; maximum loss is $350; breakeven point is $48.50.

D) BE = Breakeven is when the long stock can be sold at the customer's total cost. That cost is the price of the stock ($46) plus the price paid for the option ($2½), or $48.50. Why doesn't the breakeven follow the "put-down" rule? That rule applies when the only positions are options. Once there is a long or short stock position along with an option position, it is the stock controlling the breakeven.

If an investor has received dividends and capital gains distributions on mutual fund shares she has held for four months, she will pay A) ordinary income tax rates on the capital gains and dividends. B) no tax until she liquidates the shares. C) long-term or short-term capital gains rates, depending on the length of time the customer has held the fund shares. D) capital gains rates on capital gains distributions and ordinary income rates on dividends.

D) Capital gains distributions are taxed as capital gains, with their holding status depending on how long the fund has held the securities, not how long the investor has held the mutual fund shares. Dividend distributions are taxed as ordinary income.

All of the following actions must be completed before a customer entering her first option trade except A) completion of the new account form. B) delivery of an Options Clearing Corporation disclosure booklet. C) approval by a sales supervisor. D) completion of the options agreement.

D) Customers do not have to complete (sign) the options agreement before entering an order, although under exchange rules, the agreement must be signed and returned by the customer within 15 days of account approval.

Which of the following ratios is normally considered adequate coverage of interest and principal charges for a municipal revenue bond? A) 7.5:1 B) 3:1 C) 1:1 D) 2:1

D) Generally, a sound debt service (interest and principal) coverage ratio for municipal revenue bonds is 2:1. In other words, $2 of revenue is collected for every $1 of debt service.

The net asset value (NAV) of an international bond fund can be expected to increase if interest rates rise abroad. interest rates fall abroad. the U.S. dollar strengthens. the U.S. dollar weakens. A) II and III B) I and III C) I and IV D) II and IV

D) If interest rates fall, bond prices will rise, thus increasing the NAV of a bond portfolio. If the U.S. dollar weakens, the value of other currencies will rise. This would also increase the NAV for a portfolio of international bonds.

All of the following statements regarding a market not-held order are true except A) a small portion may be filled at a time. B) it gives the floor broker discretion over the price or time of execution. C) the order ticket must be marked, not held. D) it is given to a specialist (designated market maker).

D) In a market not-held order, the client agrees not to hold the broker responsible if she cannot fill the complete order. Such an order allows the floor broker to use her judgment on the best execution strategy. Specialists (designated market makers) cannot accept market not-held orders.

A taxable gain or loss on a long call option transaction would be recognized when the option is purchased. the option expires. the option is sold. the option is exercised. A) I and IV B) I and II C) III and IV D) II and III

D) In addition to being exercised, call options can either be sold or allowed to expire. If either of these situations occurs, the owner of the call would determine hers gain or loss (for tax purposes) at the time of expiration or sale. This would be determined by comparing what she paid for the call versus the price at which she sold the call. If it expires, the entire amount of the premium originally paid is considered a loss. Gains or losses are not determined at the time that calls are exercised. Once exercised, the underlying security must then be sold at the current market value. Then the owner of the call would calculate her profit or loss, taking into account the premium paid, what she paid for the stock, and what she subsequently sold the stock for.

An investor who makes transactions once a month using dollar cost averaging would A) buy stock in a rising market and sell stock in a falling market. B) buy the same number of shares of a stock. C) allocate assets equally among cash, stocks, and bonds. D) buy the same dollar amount of stock.

D) In the dollar cost averaging investment strategy, the number of dollars invested each month remains constant. Accordingly, the investor will automatically buy more shares when the price is low to reduce the average cost per share.

An issuer may direct sales of a new issue to all of the following except A) officers of its largest customer. B) officers of its largest supplier. C) officers of the issuer. D) officers of the managing underwriter.

D) Issuer-directed sales are permitted if the persons to whom the new issue is sold are not restricted. Officers of the managing underwriter are restricted.

Your customer, age 29, makes $42,000 annually and has $10,000 to invest. Although he has never invested before, he wants to invest in something exciting. Which of the following should you suggest? A) A growth and income fund because the customer has never invested before B) An aggressive growth fund because the customer is young and has many investing years ahead C) A balanced fund because when the stock market is declining, the bond market will perform well D) Customer should provide more information before you can make a suitable recommendation

D) It is necessary to get more information about this customer and his definitions of an exciting investment opportunity before making any recommendations. A suitability and risk-tolerance analysis should be performed before a recommendation is made.

An investor would most likely purchase money market instruments for their A) yields. B) inflation protection. C) appreciation potential. D) liquidity.

D) Money market instruments are frequently referred to as cash equivalents. That is largely due to their high liquidity. Yields on these instruments are very low, and as fixed-income instruments, they offer no appreciation potential or inflation protection.

A confirmation to a customer purchasing a new issue of municipal securities must disclose all of the following except A) customer's name. B) coupon rate and maturity date. C) settlement date. D) current yield.

D) Municipal Securities Rulemaking Board confirmations must include the customer's name, trade and settlement dates, coupon rate and maturity, and the yield to maturity or yield to call (whichever is lower). The current yield (annual interest / current market price) is not included on confirmations.

The date on which the interest on a new municipal issue begins accruing is A) the closing date. B) the settlement date. C) the delivery date. D) the dated date.

D) New issues of municipal bonds begin accruing interest on the dated date.

One of your customers would like to purchase a government agency security for the UTMA account of her daughter. The daughter worked in construction over the summer and would like to use $1,275 of her savings for the purchase. Securities issued by which of these agencies could be purchased for this account? A) Federal Farm Credit System B) Federal Home Loan Mortgage Corporation C) Student Loan Marketing Association D) Federal National Mortgage Association

D) Of this group, the only agency that would be able to sell $1,275 of securities is Fannie Mae. Their securities are available with a minimum denomination of $1,000 and then increments of $1. FHLMC also has the $1,000 initial minimum, but with $1,000 increments. The same numbers apply to the FCS, and Sallie Mae's minimum is $10,000. Another agency that would have met the investor's need is GNMA.

A customer is considering adding a real estate investment trust (REIT) to her portfolio. She lists all of the following as advantages. You correct your customer and point out that one of them is not an advantage of investing in REITs. Which of the following is not an advantage of investing in REITs? A) Being able to divest of the shares easily B) Having a professionally managed portfolio of commercial real estate assets C) Using real estate as a potential hedge against the movement of other equity securities the customer owns D) Dividend treatment

D) Of those listed, only dividend treatment can be identified as not being an advantage. While the expectation of receiving dividends is inherently good, dividends paid by REITs to their shareholders are not recognized as qualified and are therefore taxable to the investor at their full ordinary income tax rate

Under the Conduct Rules, the maximum sales charge on any transaction involving an open-end investment company share is A) 8.5% of the net asset value. B) 9% of the net asset value. C) 8.5% of the offering price. D) 9% of the offering price.

D) Open-end investment companies (mutual funds) are limited to a maximum sales charge of 8.5% of the offering price.

In determining a violation of position limits, short calls are aggregated with A) all of these. B) short puts. C) long calls. D) long puts.

D) Position limits are measured by the number of contracts on the same side of the market. Long calls and short puts are on the bull side, and short calls and long puts are on the bear side.

Regarding the sale of a new issue, a customer becomes a restricted person if he is A) a salesperson who works for a supplier of the issuing corporation. B) the grandfather of an associated person of a member firm. C) a private investigator collecting information on one of the issuing firm's officers. D) a salesperson who works for the issuing firm's underwriter.

D) Restricted persons include FINRA member firms and their associated persons, plus immediate family members.

Underwriters that reserve the right to stabilize the price of securities distributed to the public under an SEC registration statement may do so A) only if the securities being distributed will be immediately listed for trading on the NYSE or other exchange. B) under no circumstances. C) without restriction. D) only if notice is given in the prospectus.

D) Stabilizing transactions are permitted if the SEC is notified in the registration statement and the investing public is notified in the prospectus.

Which of the following governmental bodies receive the least amount of their revenues from property taxes? A) County governments B) School districts C) Municipalities D) State governments

D) State governments generally do not assess property (ad valorem) taxes. These are assessed by local governments. Generally, state governments receive most of their income from sales and income taxes.

A customer wants to buy $12,000 worth of stock using other marginable securities owned as collateral for the purchase. With Regulation T at 50%, what must the current market value of the securities deposited be? A) $6,000 B) $24,000 C) $18,000 D) $12,000

D) Stock buys stock dollar for dollar

Performance of the terms of a standardized listed option contract are guaranteed by A) the Chicago Board Options Exchange. B) FINRA. C) the Securities and Exchange Commission. D) the Options Clearing Corporation.

D) The Options Clearing Corporation issues, guarantees, and handles the exercise and assignment of listed options.

The common stock of Porcine Meat Products, Inc., is currently selling at $60 per share. It has a P/E ratio of 12:1 and pays an annual dividend of $3 per share. That would make Porcine's EPS equal to A) $3. B) $2. C) $36. D) $5.

D) The P/E ratio measures the relationship between a stock's market price and the earnings per share (EPS). The ratio for this company is 12 times the earnings. If the market price is $60, then the earnings must be 1/12th of that or $5 per share. The annual dividend is irrelevant to the question. It is one of those extra numbers that FINRA likes to include in a question.

Several years ago, an investor purchased 100 shares of XYZ stock at $50 per share. XYZ is now trading at $90. Although the investor is still bullish on the stock, there is a concern that there might be a retreat after the next earnings report is released. Which of the following options strategies would provide some downside protection at no cost to the investor? A) Buy an XYZ 90 put B) Buy an XYZ 90 call C) Write an XYZ 90 put D) Write an XYZ 90 call

D) The best downside protection would come from buying the XYZ 90 put. However, buying the put would mean paying a premium and the question specifies "at no cost." That leaves the two short choices as the only possibilities. When you are long the stock, writing a covered call produces income. The income protects the downside to the extent of the premium received. Writing a put will also generate income. However, if the stock goes up, (something the investor would like), the put will be exercised and the investor will have to buy an additional 100 shares at the strike price.

A customer buys a 20-year, 7% bond on a 7.35 basis. The bond is callable in six years at 103, in eight years at 102, in 10 years at 101, and at par beginning in the 12th year. The customer's confirmation will show yield to A) the 12-year put. B) the 10-year call. C) the 6-year call. D) the 20-year maturity.

D) The confirmation of a bond trade must disclose the lower of yield to call or yield to maturity. This bond is being bought at a discount because the basis is higher than the coupon. Because the yield to maturity on a discount bond is lower than the yield to call, this is the yield that will be shown on the confirmation.

A customer purchased 10 municipal original issue discount (OID) bonds at 92. If he holds them to maturity, he will be federally taxed on which of the following? A) $80 B) $800 C) $8,000 D) $0

D) The key here is OID. The discount on OID bonds is considered interest. A municipal bond interest is tax free. Therefore, the profit realized from the difference between the discounted purchase price and the redemption price at maturity is also considered tax free at the federal level. If the owner lives in the state within which the bonds are issued, they will be state and local tax free as well.

If a customer has a restricted margin account with special memorandum account (SMA) of $2,500, how much must he deposit to purchase $10,000 worth of stock? A) $0 B) $5,000 C) $10,000 D) $2,500

D) The purchase of $10,000 requires a $5,000 deposit, which can be reduced dollar for dollar by the existing SMA.

Which of the following is not a requirement to be included on a customer confirmation by the Municipal Securities Rulemaking Board (MSRB)? A) The amount of the dealer's markup or markdown B) Whether the trade was made as an agency transaction C) Whether the sale was made from the dealer's inventory D) The location of the trust indenture

D) The trust indenture is not required by the MSRB to be included on a customer confirmation. The official statement must include the location of the trust indenture and a statement that bondholders may review it if they choose. MSRB rules require that all confirmations include the firm's capacity in the trade (agent/principal). The amount of the dealer's markup or markdown on a principal trade must be disclosed. The commission on an agency trade must be disclosed.

Question #17 of 125 Question ID: 1282952 An investment company registered under the Investment Company Act of 1940 that allows shareholders to sell their shares back to the company at the net asset value per share only at certain specified times is A)an open-end investment company. B)a unit investment trust. C)a closed-end investment company. D)an interval fund.

D) The unique characteristic of an interval fund is that at certain specified intervals, shareholders are able to sell their shares back to the fund at NAV. True, these are closed-end investment companies, but on the exam, you always need to choose the most specific answer. Open-end investment companies (mutual funds) and unit investment trusts (UITs) permit redemption at NAV on a continuous basis, not at specified intervals.

In a margin account, the broker-dealer lends money to the customer to assist in the purchase of a marginable security. Instead of delivering the security to the purchaser, the broker-dealer holds it as collateral for the loan. The form signed by the customer agreeing to this is A) the credit agreement. B) the stock pledge agreement. C) the loan consent agreement. D) the hypothecation agreement.

D) There are three special margin account agreement forms. The hypothecation agreement is the one in which the customer agrees to allow the broker-dealer to keep the securities purchased as collateral for the margin loan. The credit agreement contains the terms of the loan, such as interest to be charged, and the loan consent agreement is an optional form agreeing to let the broker-dealer lend out those securities.

An investor buys a GO bond with a coupon of 3½% that has a basis of 3¾%. If the bond is held until maturity, the investor's actual yield will be A) more than 3¾% . B) 3½%. C) 3¾%. D) more than 3½% but less than 3¾%.

D) This is tricky, so follow along. With a coupon of 3½% and a basis (yield to maturity) of 3¾%, we know the bond was purchased at a discount. GO bonds are municipal bonds, and when a municipal bond is purchased in the secondary market at a discount, the accretion of the discount is taxed as ordinary income. Therefore, a portion of the investor's return will be taxable, making the actual return slightly less than the yield to maturity.

Capital gains distributed by a mutual fund to shareholders are reported and taxable for the year A) the shares are redeemed by the fund. B) paid by the fund. C) the shareholder chooses but not later than two years after all shares are redeemed. D) earned (accrued).

D)EARNED Capital gains can be distributed to shareholders by a mutual fund no more than once per year and are reported and taxable for the year earned (accrued).

A bond convertible at $50 is selling at 105% of parity, while the common stock has a current market value of $45. What is the market value of the bond? A) $1,000 B) $900 C) $1,045 D) $945

D) When a bond is convertible at $50, it means the holder can exchange each $1,000 par value bond for the company's common stock at a rate of $50 per share. Dividing $1,000 (always use the par value, not the market value) by $50 results in a conversion rate of 20 shares per bond. With the bond convertible into 20 shares and the market price of each share currently $45, the parity price, the price at which the value of the stock and the bond are the same, is $900, (20 x $45). The question tells us that the bond is selling for 105% of the parity price. That would be $900 x 105% = $945. An alternative method is to recognize that the stock is selling for 10% below its conversion price ($45 is $5 less than $50 and $5 ÷ $50 = 10%). That means the parity price of the bond must be 10% below the par value, or $900 (which is 10% less than $1,000). Once you have the $900, multiply by 105% to arrive at the correct answer of $945.

An investor takes a short position in one XYZ Nov 140 call @7. Disregarding any commissions, if the option is exercised, on settlement date, the investor A) receives $700. B) must pay $14,000. C) must pay $700. D) receives $14,000.

D) When an investor takes a short position in an option, it means the investor has sold, or written the option. When a call option is exercised, the seller is obligated to deliver the stock at the exercise (strike) price. A strike price of $140 for 100 shares results in the seller receiving $14,000 on settlement date.

Your customer with a discretionary account is approved to trade all options strategies and positions. Regarding discretion, the customer cautions that no positions having an unlimited loss potential should ever be used or would be acceptable to her. Which of the following options positions would still be suitable, and therefore, employed in the customer's discretionary account? A)Long and short straddles, short calls and puts B)Long calls and puts, short straddles C)Short spreads, short straddles, short calls and puts D)Long calls and puts, long straddles, long or short spreads

D) When purchasing calls or puts, loss is limited to the premium paid. When purchasing straddles, loss is limited to the combined premiums paid. When purchasing or selling either call or put spreads, the potential gain or loss is always defined—therefore, limited to some amount. These would be the only suitable positions offered: long calls and puts, long straddles, and long or short spreads. Each of the remaining choices contains a position with an unlimited loss potential such as short calls or short straddles.

A 58 year-old investor owns a single premium deferred variable annuity with a current value of $500,000. The original investment was $150,000 and the contract has a death benefit provision. If the investor suddenly passes away and the beneficiary receives a lump sum payout, A) the beneficiary will owe ordinary income taxes on $500,000. B) the beneficiary will owe ordinary income taxes on $350,000 plus 10% penalty if under 59½. C) the beneficiary will owe ordinary income taxes on $500,000 plus 10% penalty if under 59½. D) the beneficiary will owe ordinary income taxes on $350,000.

D) When receiving the death benefit in a lump sum, the beneficiary's tax situation is the same as if the owner surrendered the policy with one critical difference. Surrender before reaching age 59½ leads to the 10% tax penalty, but that penalty is waived in the case of death. It is only the deferred earnings (in this question, the $350,000 difference between the initial deposit and the current value) that is subject to taxes. As is always the case with annuities, the taxation is always as ordinary income, never capital gains.

An investor purchases a municipal bond at par to yield 5.5% to maturity. Two years later, if he sells the bonds at a price equivalent to a 5% yield to maturity, the investor incurs A) a capital gain. B) no taxable result at this time. C) tax-free income. D) a capital loss.

D) Yields fall as bond prices rise. Because the yield to maturity has dropped, the bond is trading at a higher price than when it was purchased. The consequence of the sale is a capital gain because the investor sold the bond that was purchased for par at a premium.

All of the following statements concerning IRA contributions are true except A) you may contribute to this year's IRA from January 1 of this year until April 15 of next year. B) if you file your taxes on January 15, you may deduct your IRA contribution even if it is not made until April 15. C) between January 1 and April 15, you may make contributions for the current year, the past year, or both. D) you may make contributions for the past year after April 15, provided you have filed an extension on a timely basis.

D) You may contribute to an IRA only until the first tax filing deadline (April 15) even if you filed an extension.

A customer sells securities and uses the proceeds to buy more securities at the same cost. Under the 5% markup policy, the markup is calculated on A) the sell side only. B) the buy side only. C) each side separately. D) the total of both sides.

D) a firm must consider the entire transaction when determining markup

Rule 144A regulates A) personal trading by research analysts. B) companies traded on the Nasdaq Global Select Market. C) the sale of restricted stock by control persons. D) the sale of restricted stock to institutional investors.

D) the sale of restricted stock to institutional investors known as qualified institutional buyers.

A variable-rate municipal bond investment's main advantage is that A) its interest is exempt from all taxes. B) it is likely to increase in value. C) it is noncallable. D) its price should remain relatively stable.

D)A variable-rate bond has no fixed coupon rate. The coupon is tied to a market rate (e.g., T-bond yields) and subject to change at regular intervals. Because the interest paid reflects changes in overall interest rates, the bond price remains relatively close to its par value. Its coupon is always representative of the current market rate. As rates rise, the coupon is adjusted upward. As rates fall, the coupon is adjusted downward.

A customer wanting to invest in an oil and gas limited partnership wants to know what her cost basis would be for tax purposes. While there can be a number of variables, cost basis for a limited partner (LP) is best defined as A) recourse debt minus cash contributions. B) noncash contribution plus nonrecourse debt minus recourse debt. C) cash investment made minus distributions. D) cash investment made plus recourse debt minus distributions.

D)Cost basis for a limited partner is defined as investment made (cash contributions) plus recourse debt (debt the LP is responsible for) minus distributions. Nonrecourse debt would only be included for real estate programs. Real estate programs are the only types where LPs can be responsible for both recourse and nonrecourse debt.

Most mutual funds operate as regulated investment companies. This means that A) their principal underwriter (sponsor) is a FINRA member. B) they register with the SEC under the Investment Advisers Act of 1940. C) they register with the SEC under the Investment Company Act of 1940. D) they qualify for special tax treatment under Subchapter M of the Internal Revenue Code.

D)Triple taxation of investment income can be avoided if the mutual fund qualifies under Subchapter M of the IRC. To avoid taxation under Subchapter M, a fund must distribute at least 90% of its net investment income to shareholders. The fund then pays taxes only on the undistributed amount. This rule applies to management companies (open-end and closed-end) and UITs. That means ETFs are also included. Although not investment companies registered under the Investment Company Act of 1940, REITs can also take advantage of Subchapter M's tax benefits.

NYSE AND NASDAQ

Do not accept GTC or stop orders or AON or FOK

Income from which of the following investments is passive income? Real estate direct participation programs (DPPs) Vacation cottage rentals Real estate investment trusts (REITs) Collateralized mortgage obligations (CMOs)

Passive income results from DPPs and personal real estate rentals. REITs and CMOs are securities, and income from securities is considered portfolio income.

Which of the following bonds is issued to finance the construction of subsidized housing and is backed by rents and the taxing authority of the U.S. government? A)Moral obligation B)Special tax C)Section 8 D)Special assessment

Section 8 bonds, also known as Public Housing Authority bonds and New Housing Authority bonds, are used to finance subsidized housing. These bonds are backed by rental income. If this income is insufficient to service the debt, the U.S. government makes up the difference. Essentially, these are revenue bonds backed by the U.S. government, and are therefore AAA rated.

When a registered representative opens an options account for a new client, in which order must the following actions take place? Obtain approval from the branch manager Obtain essential facts from the customer Obtain a signed options agreement Enter the initial order

The steps in opening a new options account occur in the following order: obtain essential facts about the customer, have the manager approve the account, enter the initial order, and have the customer sign the options agreement within 15 days. What about delivery of the options disclosure document (ODD)? That isn't included in the choices here. It must be delivered at or prior to the time the account is approved for options trading. That would mean before or simultaneously with choice I.


Conjuntos de estudio relacionados

Prep U Chapter 34: Assessment and Management of Patients with Inflammatory Rheumatic Disorders

View Set

Mod 08: Nation Building in the Americas

View Set

Managing Anxiety and Delivering your Speech

View Set

IB Economics SL - T2: Macroeconomics

View Set